Help Pleasss I will give brainlyest!!!! :D

Help Pleasss I Will Give Brainlyest!!!! :D

Answers

Answer 1

Answer: The answer is 2,4. May I have the brainiest? pls I only need one more.


Related Questions

why infinity ( ) can’t be included in an inequality?

Answers

Answer:

Step-by-step explanation:

Because then the value on the other side will be unbounded by the infinity sign while expressing the answers on a number line.

please click thanks and mark brainliest if you like :)

Illustrate the 7th pattern of the sequence of square numbers. ​

Answers

1,4,9,16,25,36,49,........

7th pattern =49.....

Answer:

1, 4, 9, 16, 25, 36, 49…................the 7 the pattern is 49

what is the slope and point

Answers

Answer:

Step-by-step explanation:

A baseball team plays in a stadium that holds 58000 spectators. With the ticket price at $12 the average attendance has been 25000. When the price dropped to $9, the average attendance rose to 29000. Assume that attendance is linearly related to ticket price.

Required:
a. Find the demand function p(x), where x is the number of the spectators.
b. How should ticket prices be set to maximize revenue?

Answers

Answer:

We need to assume that the relationship is linear.

a) Remember that a linear relation is written as:

y = a*x + b

then we will have:

p(x) = a*x + b

where a is the slope and b is the y-intercept.

If we know that the line passes through the points (a, b) and (c, d), then the slope can be written as:

y = (d - b)/(c - a)

In this case, we know that:

if the ticket has a price of $12, the average attendance is 25,000

Then we can define this with the point:

(25,000 , $12)

We also know that when the price is $9, the attendance is 29,000

This can be represented with the point:

(29,000, $9)

Then we can find the slope as:

a = ($9 - $12)/(29,000 - 25,000) = -$3/4,000 = -$0.00075

Then the equation is something like:

y = (-$0.00075)*x + b

to find the value of b we can use one of the known points.

For example, the point (25,000 , $12) means that when x = 25,000, the price is $12

then:

$12 = (-$0.00075)*25,000 + b

$12 = -$18.75 + b

$12 + $18.75 = b

$30.75 = b

Then the equation is:

p(x) = (-$0.00075)*x + $30.75

b) We want to find the ticket price such that it maximizes the revenue.

The revenue will be equal to the price per ticket, p(x) times the total attendance, x.

Then the revenue can be written as:

r(x) = x*p(x) = x*( (-$0.00075)*x + $30.75 )

r(x) =  (-$0.00075)*x^2 + $30.75*x

So we want to find the maximum revenue.

Notice that this is a quadratic equation with a negative leading coefficient, thus the maximum will be at the vertex.

Remember that for an equation like:

y = a*x^2 + bx + c

the x-value of the vertex is:

x = -b/2a

Then in our case, the x-value will be:

x = -$30.75/(2*(-$0.00075)) = 20,500

Then the revenue is maximized for x = 20,500

And the price for this x-vale is given by:

p( 20,500) =  (-$0.00075)*20,500 + $30.75 = $15.375

which should be rounded to $15.38

A driveway is 2/5 kilometers long. Omar walks 1/3 of the driveway. How far does Omar walk?

Answers

Answer:

2/15 kilometers

Step-by-step explanation:

Multiply the distance by the fraction walked

2/5 kilometers * 1/3

2/5 * 1/3

2/15 kilometers

A.) V’ (-3,-5), K’ (-1,-2), B’ (3,-1), Z’(2,-5)

B.) V’(-4, 1), K’(-2, 4), B(2,5) Z’ (1, 1)

C.) V’ (-3,-4), K’(-1,-1) B’ (3,0), Z’(2,-4)

D.) V’ (-1,0), K’ (1, 3), B’(5,4), Z’(4,0)

Answers

Answer:

C

Step-by-step explanation:

this is a "translation" - a shift of the object without changing its shadow and size.

this shift is described by a "vector" - in 2D space by the x and y distances to move.

we have here (1, 0) - so, we move every point one unit to the right (positive x direction) and 0 units up/down.

therefore, C is the right answer (the x coordinates of the points are increased by 1, the y coordinate are unchanged).

Use the information below to complete the problem: p(x)=1/x+1 and q(x)=1/x-1 Perform the operation and show that it results in another rational expression. p(x) + q(x)

Answers

Answer:

hope u will understand...if u like this answer plz mark as brainlist

Answer:

[tex]\displaystyle p(x) + q(x) = \frac{2x}{(x+1)(x-1)}[/tex]

The result is indeed another rational expression.

Step-by-step explanation:

We are given the two functions:

[tex]\displaystyle p(x) = \frac{1}{x+1}\text{ and } q(x) = \frac{1}{x-1}[/tex]

And we want to perform the operation:

[tex]\displaystyle p(x) + q(x)[/tex]

And show that the result is another rational expression.

Add:

[tex]\displaystyle = \frac{1}{x+1} + \frac{1}{x-1}[/tex]

To combine the fractions, we will need a common denominator. So, we can multiply the first fraction by (x - 1) and the second by (x + 1):

[tex]\displaystyle = \frac{1}{x+1}\left(\frac{x-1}{x-1}\right) + \frac{1}{x-1}\left(\frac{x+1}{x+1}\right)[/tex]

Simplify:

[tex]=\displaystyle \frac{x-1}{(x+1)(x-1)} + \frac{x+1}{(x+1)(x-1)}[/tex]

Add:

[tex]\displaystyle = \frac{(x-1)+(x+1)}{(x+1)(x-1)}[/tex]

Simplify. Hence:

[tex]\displaystyle p(x) + q(x) = \frac{2x}{(x+1)(x-1)}[/tex]

The result is indeed another rational expression.

Translate this phrase into an algebraic expression.
61 less than twice Jenny's age
Use the variable j to represent Jenny's age.

Answers

ANSWER: 2j-61
j = Jenny's age

find the area of the triangle

Answers

9514 1404 393

Answer:

  108 cm²

Step-by-step explanation:

The area of a triangle is given by the formula ...

  A = 1/2bh

where b represents the base length, and h represents the height--the perpendicular distance from the base to the opposite vertex. The area of this triangle is ...

  A = 1/2(12 cm)(18 cm) = 108 cm²

Which equation is represented by the graph below?

Answers

Answer:

Hello,

Answer C

Step-by-step explanation:

Since ln(1)=0

if x=1 then y=4 ==> y=ln(x)+4

y=ln(x) is translated up for 4 units.

Please help!
What is the pattern,
Y-interception
And equation

Answers

Answer: y=1x+1

Step-by-step explanation:

y=1x+3

that should be it

WORTH 100 POINTS!
The function h(x) is quadratic and h(3) = h(-10) = 0. Which could represent h(x)?

1) h(x) = x2 - 13x - 30
2) h(x) = x2 - 7x - 30
3) h(x) = 2x2 + 26x - 60
4) h(x) = 2x2 + 14x - 60

Answers

Answer:

h(x) = 2x^2 +14x -60

Step-by-step explanation:

A quadratic is of the form

h(x) = ax^2 + bx +c

h(3) = h(-10) = 0

This tells us that the zeros are at x=3 and x = -10

We can write the equation in the form

h(x) = a( x-z1)(x-z2) where z1 and z2 are the zeros

h(x) = a(x-3) (x- -10)

h(x) = a(x-3) (x+10)

FOIL

h(x) = a( x^2 -3x+10x-30)

h(x) = a(x^2 +7x -30)

Let a = 2

h(x) = 2x^2 +14x -60

It means

zeros are 3 and -10

Form equation

y=x²-(3-10)x+(-10)(3)y=x²+7x-30

Multi ply by 2

y=2x²+14x-60

Option D

Please answer this question

Answers

The answer is C. 4.1¯6

If per unit variable cost of a product is Rs.8 and fixed cost is Rs 5000 and it is sold for Rs 15 per unit, profit in 1000 units is.......
a.. rs 7000
b. rs 2000
c. rs 25000
d. rs 0​

Answers

Answer:

a.. rs 7000

Because 15×1000=15000 it is SP when selling 1000units in the rate of Rs 15/unit& 8×1000=8000 this is cp when buying 1000 units in the rate of Rs 8/unit.

So,by formula of profit,

Rs (15000-8000)=Rs7000

10. (30-i)-(18+6i)+30i

Answers

Answer:

[tex]12+23i[/tex]

Step-by-step explanation:

[tex](30−i)−(18+6i)+30i[/tex]

[tex]30−i−18−6i+30i[/tex]

[tex]12−i−6i+30i[/tex]

[tex]12−7i+30i[/tex]

[tex]12+23i[/tex]

Hope it is helpful....

Find the sum of ∑3/k=0 k^2

Answers

Answer:

[tex]14[/tex]

Step-by-step explanation:

Given

[tex]\displaystyle \sum_{k=0}^3k^2[/tex]

Let's break down each part. The input at the bottom, in this case [tex]k=0[/tex], is assigning an index [tex]k[/tex] at a value of [tex]0[/tex]. This is the value we should start with when substituting into our equation.

The number at the top, in this case 3, indicates the index we should stop at, inclusive (meaning we finish substituting that index and then stop). The equation on the right, in this case [tex]k^2[/tex], is the equation we will substitute each value in. After we substitute our starting index, we'll continue substituting indexes until we reach the last index, then add up each of the outputs produced.

Since [tex]k=0[/tex] is our starting index, start by substituting this into [tex]k^2[/tex]:

[tex]0^2=0[/tex]

Now continue with [tex]k=1[/tex]:

[tex]1^1=1[/tex]

Repeat until we get to the ending index, [tex]k=3[/tex]. Remember to still use [tex]k=3[/tex] before stopping!

Substituting [tex]k=2[/tex]:

[tex]2^2=4[/tex]

Substituting [tex]k=3[/tex]:

[tex]3^2=9[/tex]

Since 3 is the index we end at, we stop here. Now we will add up each of the outputs:

[tex]0+1+4+9=\boxed{14}[/tex]

Therefore, our answer is:

[tex]\displaystyle \sum_{k=0}^3k^2=0+1+4+9=\boxed{14}[/tex]

Answer:

14

Step-by-step explanation:

∑3/k=0 k^2

Let k=0

0^2 =0

Let k = 1

1^2 =1

Let k =2

2^2 = 4

Let k = 3

3^2 = 9

0+1+4+9 = 14

help with 1 b please. using ln.​

Answers

Answer:

[tex]\displaystyle \frac{dy}{dx} = \frac{1}{(x - 2)^2\sqrt{\frac{x}{2 - x}}}[/tex]

General Formulas and Concepts:

Pre-Algebra

Equality Properties

Algebra I

Terms/CoefficientsFactoringExponential Rule [Root Rewrite]:                                                                 [tex]\displaystyle \sqrt[n]{x} = x^{\frac{1}{n}}[/tex]

Algebra II

Natural logarithms ln and Euler's number eLogarithmic Property [Exponential]:                                                             [tex]\displaystyle log(a^b) = b \cdot log(a)[/tex]

Calculus

Differentiation

DerivativesDerivative NotationImplicit Differentiation

Derivative Property [Multiplied Constant]:                                                           [tex]\displaystyle \frac{d}{dx} [cf(x)] = c \cdot f'(x)[/tex]

Derivative Property [Addition/Subtraction]:                                                         [tex]\displaystyle \frac{d}{dx}[f(x) + g(x)] = \frac{d}{dx}[f(x)] + \frac{d}{dx}[g(x)][/tex]

Basic Power Rule:

f(x) = cxⁿf’(x) = c·nxⁿ⁻¹

Derivative Rule [Quotient Rule]:                                                                           [tex]\displaystyle \frac{d}{dx} [\frac{f(x)}{g(x)} ]=\frac{g(x)f'(x)-g'(x)f(x)}{g^2(x)}[/tex]

Derivative Rule [Chain Rule]:                                                                                 [tex]\displaystyle \frac{d}{dx}[f(g(x))] =f'(g(x)) \cdot g'(x)[/tex]

Step-by-step explanation:

*Note:

You can simply just use the Quotient and Chain Rule to find the derivative instead of using ln.

Step 1: Define

Identify

[tex]\displaystyle y = \sqrt{\frac{x}{2 - x}}[/tex]

Step 2: Rewrite

[Function] Exponential Rule [Root Rewrite]:                                               [tex]\displaystyle y = \bigg( \frac{x}{2 - x} \bigg)^\bigg{\frac{1}{2}}[/tex][Equality Property] ln both sides:                                                                 [tex]\displaystyle lny = ln \bigg[ \bigg( \frac{x}{2 - x} \bigg)^\bigg{\frac{1}{2}} \bigg][/tex]Logarithmic Property [Exponential]:                                                             [tex]\displaystyle lny = \frac{1}{2}ln \bigg( \frac{x}{2 - x} \bigg)[/tex]

Step 3: Differentiate

Implicit Differentiation:                                                                                 [tex]\displaystyle \frac{dy}{dx}[lny] = \frac{dy}{dx} \bigg[ \frac{1}{2}ln \bigg( \frac{x}{2 - x} \bigg) \bigg][/tex]Logarithmic Differentiation [Derivative Rule - Chain Rule]:                       [tex]\displaystyle \frac{1}{y} \ \frac{dy}{dx} = \frac{1}{2} \bigg( \frac{1}{\frac{x}{2 - x}} \bigg) \frac{dy}{dx} \bigg[ \frac{x}{2 - x} \bigg][/tex]Chain Rule [Basic Power Rule]:                                                                     [tex]\displaystyle \frac{1}{y} \ \frac{dy}{dx} = \frac{1}{2} \bigg( \frac{1}{\frac{x}{2 - x}} \bigg) \bigg[ \frac{2}{(x - 2)^2} \bigg][/tex]Simplify:                                                                                                         [tex]\displaystyle \frac{1}{y} \ \frac{dy}{dx} = \frac{-1}{x(x - 2)}[/tex]Isolate  [tex]\displaystyle \frac{dy}{dx}[/tex]:                                                                                                     [tex]\displaystyle \frac{dy}{dx} = \frac{-y}{x(x - 2)}[/tex]Substitute in y [Derivative]:                                                                           [tex]\displaystyle \frac{dy}{dx} = \frac{-\sqrt{\frac{x}{2 - x}}}{x(x - 2)}[/tex]Rationalize:                                                                                                     [tex]\displaystyle \frac{dy}{dx} = \frac{-\frac{x}{2 - x}}{x(x - 2)\sqrt{\frac{x}{2 - x}}}[/tex]Rewrite:                                                                                                         [tex]\displaystyle \frac{dy}{dx} = \frac{-x}{x(x - 2)(2 - x)\sqrt{\frac{x}{2 - x}}}[/tex]Factor:                                                                                                           [tex]\displaystyle \frac{dy}{dx} = \frac{-x}{-x(x - 2)^2\sqrt{\frac{x}{2 - x}}}[/tex]Simplify:                                                                                                         [tex]\displaystyle \frac{dy}{dx} = \frac{1}{(x - 2)^2\sqrt{\frac{x}{2 - x}}}[/tex]

Topic: AP Calculus AB/BC (Calculus I/I + II)

Unit: Differentiation

Book: College Calculus 10e

if point B is the midpoint of points A and C, find the value of x and AC. AB= 5x - 2, BC= 9x -10

Answers

9514 1404 393

Answer:

x = 2AC = 16

Step-by-step explanation:

The midpoint divides the segment into two equal lengths:

  AB = BC

  5x -2 = 9x -10

  8 = 4x

  2 = x

  AB = 5(2) -2 = 8

  AC = 2AB = 2(8) = 16

We are given a weighted coin (with one side heads, one side tails), and we want to estimate the unknown probability pp that it will land heads. We flip the coin 1000 times and it happens to land heads 406 times. Give answers in decimal form, rounded to four decimal places (or more). We estimate the chance this coin will land on heads to

Answers

Answer:

0.4060

Step-by-step explanation:

To calculate the sample proportion, phat, we take the ratio of the number of preferred outcome to the total number of trials ;

Phat = number of times coin lands on head (preferred outcome), x / total number of trials (total coin flips), n

x = 406

n = 1000

Phat = x / n = 406/ 1000 = 0.4060

The estimate of the chance that this coin will land on heads is 0.406

Probability is the likelihood or chance that an event will occur.

Probability = Expected outcome/Total outcome

If a coin is flipped 1000 times, the total outcomes will 1000

If it landed on the head 406 times, the expected outcome will be 406.

Pr(the coin lands on the head) = 406/1000

Pr(the coin lands on the head) = 0.406

Hence the estimate of the chance that this coin will land on heads is 0.406

Learn more on probability here: https://brainly.com/question/14192140

if sin150=1/2 then find sin75

Answers

Answer:

0.966

Step-by-step explanation:

When typed into a calculator, sin75 = -0.3877816354

Upon converting to degrees, the full answer is 0.96592582628

Again need help with these ones I don’t understand and they have to show work

Answers

Let’s rewrite the given equation by adding 81 to both sides:
[tex]x^2 - 18x + 81= 65 + 81[/tex]
[tex](x - 9)^2 = 146[/tex]
Taking the square root of both sides, we get
[tex]x - 9 = \pm\sqrt{146}[/tex]
or
[tex]x = 9 \pm \sqrt{146} = 9 \pm 12.1 = 21.1\:\text{and}\:-3.1[/tex]

You flip a coin that is not fair, the prbability of heads on each flip is 0.7. if the coin shows heads, you draw a marble from urn h with 1 blue and 4 red marbles. if the coin shows tails, you draw a marble from urn t with 3 blue and 1 red marble. Find the following probabilities:

a. The probability of choosing a red marble.
b. The probability of choosing a blue marble, given that the coin showed heads.
c. The probability that the coin showed tails, given that the marble was red.

Answers

Solution :

P(H) = 0.7  ;  P(T) = 0.3

If heads, then Urn H,  1 blue and 4 red marbles.

If tails, then Urn T ,  3 blue and 1 red marbles.

a).

P ( choosing a Red marble )

= P (H) x P( Red from Urn H) + P (T) x P( Red from Urn T)

[tex]$=0.7 \times \frac{4}{5} + 0.3 \times \frac{1}{4}$[/tex]

= 0.56 + 0.075

= 0.635

b).  If P (B, if coin showed heads)

If heads, then marble is picked from Urn H.

Therefore,

P (Blue) [tex]$=\frac{1}{5}$[/tex]

             = 0.2

c). P (Tails, if marble was red)

[tex]$=P (T/R) = \frac{P(R/T)}{P(R)} \ P(T)$[/tex]

Where P (R/T) = P ( red, if coin showed tails)

                        [tex]$=\frac{1}{4}$[/tex]

                        = 0.25 (As Urn T is chosen)

P (R) =  P (Red) = 0.635 (from part (a) )

P (T) = P (Tails) = 0.3

∴ [tex]$P(T/R) = \frac{0.25 \times 0.3}{0.635}$[/tex]

                = 0.118

                 

Which expression defines the given series for seven terms?

–4 + (–5) + (–6) + . . .

Answers

Answer: -n+(-n-1)

Step-by-step explanation:

Expression will be -n + (-1)

Series

-4 +(-5)+(-6)+(-7)+(-8)+(-9)+(-10)+(-11)+(-12)+(-13) and so on

Here number -n has + (-n-1) being added to it

please click thanks and mark brainliest if you like :)

Addition prop of equality
subtraction prop of quality
multiplication prop of equality
Division prop of equality
simplifying
distrib prop

Answers

1 multiplication prop
2simplifying
3 Addition prop
4 simplifying

Polynomials with odd degrees typically make a "u-shaped graph" and polynomials with even degrees typically make an "s-shaped" graph.
True
False

Answers

the answer is True :)

The statement that odd degree polynomials have a u-shaped graph and even degree polynomials have an s-shaped graph is FALSE.

What do odd degree polynomials look like on a graph?

Odd degree polynomials have branches that go in opposing directions which means that they will form an s-shaped graph.

Even degree polynomials on the other hand, have graphs that go in the same direction which is why they form u-shaped graphs.

In conclusion, the above statement is false.

Find out more on polynomials at https://brainly.com/question/9696642.

what decimal is equivalent to 0.85

Answers

Answer: 17/20

Step-by-step explanation:

0.85 = 85/100 = 17/20

The number 0.85 can be written using the fraction 85/100 which is equal to 17/20 when reduced to lowest terms.

12) Find the angles between 0o and 360o where sec θ = −3.8637 . Round to the nearest 10th of a degree:

Please show all work

Answers

9514 1404 393

Answer:

  105.0°, 255.0°

Step-by-step explanation:

Many calculators do not have a secant function, so the cosine relation must be used.

  sec(θ) = -3.8637

  1/cos(θ) = -3.8637

  cos(θ) = -1/3.8637

  θ = arccos(-1/3.8637) ≈ 105.000013°

The secant and cosine functions are symmetrical about the line θ = 180°, so the other solution in the desired range is ...

  θ = 360° -105.0° = 255.0°

The angles of interest are θ = 105.0° and θ = 255.0°.

use undetermined coefficient to determine the solution of:y"-3y'+2y=2x+ex+2xex+4e3x​

Answers

First check the characteristic solution: the characteristic equation for this DE is

r ² - 3r + 2 = (r - 2) (r - 1) = 0

with roots r = 2 and r = 1, so the characteristic solution is

y (char.) = C₁ exp(2x) + C₂ exp(x)

For the ansatz particular solution, we might first try

y (part.) = (ax + b) + (cx + d) exp(x) + e exp(3x)

where ax + b corresponds to the 2x term on the right side, (cx + d) exp(x) corresponds to (1 + 2x) exp(x), and e exp(3x) corresponds to 4 exp(3x).

However, exp(x) is already accounted for in the characteristic solution, we multiply the second group by x :

y (part.) = (ax + b) + (cx ² + dx) exp(x) + e exp(3x)

Now take the derivatives of y (part.), substitute them into the DE, and solve for the coefficients.

y' (part.) = a + (2cx + d) exp(x) + (cx ² + dx) exp(x) + 3e exp(3x)

… = a + (cx ² + (2c + d)x + d) exp(x) + 3e exp(3x)

y'' (part.) = (2cx + 2c + d) exp(x) + (cx ² + (2c + d)x + d) exp(x) + 9e exp(3x)

… = (cx ² + (4c + d)x + 2c + 2d) exp(x) + 9e exp(3x)

Substituting every relevant expression and simplifying reduces the equation to

(cx ² + (4c + d)x + 2c + 2d) exp(x) + 9e exp(3x)

… - 3 [a + (cx ² + (2c + d)x + d) exp(x) + 3e exp(3x)]

… +2 [(ax + b) + (cx ² + dx) exp(x) + e exp(3x)]

= 2x + (1 + 2x) exp(x) + 4 exp(3x)

… … …

2ax - 3a + 2b + (-2cx + 2c - d) exp(x) + 2e exp(3x)

= 2x + (1 + 2x) exp(x) + 4 exp(3x)

Then, equating coefficients of corresponding terms on both sides, we have the system of equations,

x : 2a = 2

1 : -3a + 2b = 0

exp(x) : 2c - d = 1

x exp(x) : -2c = 2

exp(3x) : 2e = 4

Solving the system gives

a = 1, b = 3/2, c = -1, d = -3, e = 2

Then the general solution to the DE is

y(x) = C₁ exp(2x) + C₂ exp(x) + x + 3/2 - (x ² + 3x) exp(x) + 2 exp(3x)

Find the multiplicative inverse of: -3/7 X -4/9

Answers

Hi there!  

»»————- ★ ————-««

I believe your answer is:  

[tex]\frac{21}{4}[/tex]

»»————- ★ ————-««  

Here’s why:  

⸻⸻⸻⸻

[tex]\boxed{\text{Calculating the answer...}}\\\\---------------\\\rightarrow -\frac{3}{7} * -\frac{4}{9}\\\\\rightarrow \frac{12}{63} \\\\\rightarrow \frac{12/3}{63/3}\\\\\rightarrow\boxed{\frac{4}{21}}\\--------------\\\rightarrow \frac{4}{21}* x= 1\\\\\rightarrow (21)*\frac{4}{21}x= 1(21)\\\\\rightarrow 4x=21\\\\\rightarrow \frac{4x=21}{4}\\\\\rightarrow \boxed{x=\frac{21}{4}}[/tex]

»»————- ★ ————-««  

Hope this helps you. I apologize if it’s incorrect.  

Subtract the integers. 22−​(−10​)​

Answers

Answer:

32

Step-by-step explanation:

Step 1: change 22 - ( -  10) into 22 + 10

Step 2: solve it like normal

Other Questions
Simplify : -2a(a-d)+2a(2a-3D) pls help. plz fast for 10 points don't scam plz plz Which of the following was not a factor in the historical lack of special "child" status, distinct from adult status? A. Infant mortality rates B. The practice of wet-nursing C. Inability to provide economically for the young D. Short life expectancies Carboxylic acid derivatives undergo hydrolysis to make carboxylic acids.a. Trueb. False Cassie has 28 days to complete a packet. The packet has 150 questions in it. How many questions a day would she need to answer to get it completed in the time frame? what is a paradox ? is silence a good or a bad thing ? give reason . plz answer Convert the credit card rate to the APR. Ohio, 0.02192% daily rate. Please round your answer to the nearest percent. (zero decimal place.) Enter only the number without $ sign. Need intoduction on this topic for argumentative essay..... ASAPparent has no right to control children above 16. Don't suggest that I am antidemocratic or somehow un-American. I'm a true patriotwho believes that the best public policy is a product of what the ____________ of American citizens prefer.a. best educatedb. majorityc. minority Production possibilities curves are useful to businesses because theyO A. help make predictions about which products will be profitable.O B. show how producing one product affects production of another.O C. compare productivity between multiple companies.O D. identify efficient production methods to increase productivity.SUBMIPREVIOUS Lament has a jar containing 6 red chips, 10 blue chips, and 4 yellow chips. If he removes one chip at random, what is the probability that it will not be red? Shortening clauses to phrases While children are playing at recess, they may not push or shove eachother. Skysong Company is considering two capital investment proposals. Estimates regarding each project are provided below: Project Soup Project Nuts Initial investment $400000 $600000 Annual net income 18000 34000 Net annual cash inflow 96000 119000 Estimated useful life 5 years 6 years Salvage value 0 0 The company requires a 10% rate of return on all new investments. Present Value of an Annuity of 1 Periods 9% 10% 11% 12% 5 3.89 3.791 3.696 3.605 6 4.486 4.355 4.231 4.111 The annual rate of return for Project Soup is Plz help me I need the answer by today. Among various ethnic groups, the standard deviation of heights is known to be approximately three inches. We wish to construct a 95% confidence interval for the mean height of male Swedes. Forty-eight mile Swedes are surveyed. The sample mean is 71 inches. The sample standard deviation is 2.8 inches.Required:a. Calculate the error bound.b. What will happen to the level of confidence obtained if 1,000 male Swedes are surveyed instead of 48? Why? Which is a graph of g(c) = (0.5)x+3^ -4 Choose the correct elements in the set for the following:{y | y is an integer and y >/= -3}{3, 4, 5, 6, . . .}{2, 1, 0, 2, . . .}{1, 0, 1, 2, . . }{3, 2, 1, 0, . . .}****PLEASE explain your answer**** Which of the following is anexample of facilitateddiffusion?A. liquid molecules colliding with oneanotherB. reaching an equal amount of molecules onboth side of a membraneC. red blood cells allowing glucose to move inand outD. cytoplasm keeping an organelle in place Hy m t cc bc to th vin cDNA t t bo mu heo. what is meant by specific latent heat of vaporization of water is -2.26mjkg^-1 or -2.26mj/kg?